Legal commentator: The goal of a recently enacted law that bans smoking in workplaces is to protect employees from se...

AndrewArabie on July 20, 2022

Why not A?

The question prompt says the law is written in such a way that it cannot be interpreted to prevent people from smoking inside their home. Assuming that is true from the stimulus, "The statements above, if true..." why wouldn't the answer be A if we are to assume the facts in the prompt are inarguable?

Reply
Create a free account to read and take part in forum discussions.

Already have an account? log in

Emil-Kunkin on July 22, 2022

Hi Andrew,

We are not actually told if law was intended to prevent people from smoking in their own homes, or if the lawmakers did not even consider that. We only know that the goal was to ban smoking in workplaces. So, we cannot know if the interpretation will be consistent with the intentions, since the only thing we know about the intent was that is was the protect employees, and the only think we know about the interpretation will not ban smoking in homes.